You are on page 1of 9

Solutions to Peskin and Schroder – Andrzej Pokraka

Problem 11.2: A zeroth-order natural relation.


This problem studies an N = 2 linear sigma model coupled to fermions:
1 1 λ
L= (∂µ φi )2 + µ2 (φi )2 − ((φi )2 )2 + ψ̄(i∂ )ψ − g ψ̄(φ1 + iγ 5 φ2 )ψ (1)
2 2 4
where φi is a two-component field, i = 1, 2.
(a) Show that this theory has the following global symmetry:

φ1 → cos αφ1 − sin αφ2 ,


φ2 → sin αφ1 + cos αφ2 ,
5
ψ → e−iαγ /2
ψ. (2)

Show also that the solution to the classical equations of motion with the minimum energy
breaks this symmetry spontaneously.
(b) Denote the vacuum expectation value of the field φi by v and make the change of variables

φi (x) = (v + σ(x), π(x)) . (3)

Write out the Lagrangian in these new variables, and show that the fermion acquires a
mass given by
mf = gv. (4)

(c) Compute the one-loop radiative correction to mf , choosing renormalization conditions so


that v and g (defined as the ψψπ vertex at zero momentum transfer) receive no radiative
corrections. Show that relation (4) receives nonzero corrections but that these corrections
are finite. This is in accord with our general discussion in Section 11.6.

Part (a)
To get a better understanding of what (1) describes we expand the sums to get

L = L0 + Lint ,
1  1
(∂µ φ1 )2 + µ2 (φ1 )2 + (∂µ φ2 )2 + µ2 (φ2 )2 + ψ̄(i∂ )ψ,

L0 =
2 2
λ 1 4 λ 2 4 λ 1 2 2 2
Lint = − (φ ) − (φ ) − (φ ) (φ ) − gφ1 ψ̄ψ − igφ2 ψ̄γ 5 ψ. (5)
4 4 2
From the above equation we see that L describes two real scalar φ4 fields that can interact with
each other and a massless fermion field.
To show that equation (2) is indeed a symmetry of the Lagrangian (1), it is easiest to
use the unexpanded Lagrangian of (1). The vector dot product (φi )2 is invariant under the
transformation (2)

(φi )2 = (φ1 )2 + (φ2 )2


→ cos2 α(φ1 )2 + sin2 α(φ2 )2 − 2 sin α cos αφ1 φ2
+ sin2 α(φ1 )2 + cos2 α(φ2 )2 + 2 sin α cos αφ1 φ2
= (φ1 )2 + (φ2 )2 . (6)

This means that the first three terms of (1) are invariant under this symmetry. Next, we show
that the fermionic kinetic term is invariant under this transformation
5 5 5
/2 −iαγ 5 /2
ψ̄i∂ ψ → ψ̄e−iαγ /2
i∂ e−iαγ /2
ψ = ψ̄i∂ e+iαγ e ψ = ψ̄i∂ ψ. (7)

1
Solutions to Peskin and Schroder – Andrzej Pokraka

Finally, we show that the fermion-scalar interactions are invariant under this transformation
5 5
φ1 ψ̄ψ + iφ2 ψ̄γ 2 ψ → cos αφ1 − sin αφ2 ψ̄e+iαγ /2 e−iαγ /2 ψ

5 5
+ i sin αφ1 + cos αφ2 ψ̄e−iαγ /2 γ 5 e−iαγ /2 ψ

5 5
= cos αφ1 ψ̄e−iαγ ψ + i sin αφ1 ψ̄γ 5 e−iαγ ψ
5 5
− sin αφ2 ψ̄e−iαγ ψ + i cos αφ2 ψ̄γ 5 e−iαγ ψ
5
= φ1 ψ̄ cos α + i sin αγ 5 e−iαγ ψ

5
+ iφ2 ψ̄γ 5 cos α + i sin αγ 5 e−iαγ ψ

5 5 5 5
= φ1 ψ̄e+iαγ e−iαγ ψ + iφ2 ψ̄γ 5 e+iαγ e−iαγ ψ
= φ1 ψ̄ψ + iφ2 ψ̄γ 5 ψ. (8)

Now we want to show that the classical solutions to the equation of motion that minimize
the energy break the symmetry (2). The Hamiltonian density is given by
X L L L
H= (∂ φi ) +
i) 0
(∂0 ψ̄) + (∂0 ψ) − L
i=1,2
∂(∂0 φ ∂(∂ 0 ψ̄) ∂(∂0 ψ)

1 1 λ
= (∂0 φ1 )2 + (∇φ1 )2 − µ2 (φ1 )2 + (φ1 )4
2 2 4
1 1 λ
+ (∂0 φ2 ) + (∇φ2 ) − µ (φ2 )2 + (φ2 )4
2 2 2
2 2 4
λ
+ (φ1 )2 (φ2 )2 + iψ̄(γ · ∇)ψ + gφ1 ψ̄ψ + igφ2 ψ̄γ 5 ψ (9)
2
Ignoring the fermion part of the Hamiltonian, the Hamiltonian is minimized by the uniform
fields, φi0 that minimize the potential

1 λ
V (φ) = − µ2 (φi )2 + ((φi )2 )2 . (10)
2 4
Taking the first derivative with respect to φi we obtain the condition that φi0 must satisfy

∂V (φ)
= −µ2 φi0 + λ(φi0 )2 φi0 = 0
∂φi φi
0

µ2
=⇒ (φi0 )2 = ≡ v. (11)
λ
How can this be done without setting g = 0? How does the Fermion fields effect v and
spontaneous symmetry breaking?

2
Solutions to Peskin and Schroder – Andrzej Pokraka

Part (b)
We denote the vacuum expectation value of φi by v and make the change of variables φ1 → v + σ
and φ2 → π. In terms of the new fields σ and π the Lagrangian is
1  1
(∂µ (v + σ))2 + µ2 (v + σ)2 + (∂µ π)2 + µ2 (π)2 + ψ̄(i∂ )ψ

L=
2 2
λ λ λ
− (v + σ) − (π) − (v + σ)2 (π)2 − g(v + σ)ψ̄ψ − igπ ψ̄γ 5 ψ
4 4
4 4 2
1  1
(∂µ σ) + µ (σ + 2vσ + v 2 ) +
2 2 2
(∂µ π)2 + µ2 π 2 + ψ̄(i∂ )ψ

=
2 2
λ 4 λ λ
− (σ + 4vσ + 6v σ + 4v σ + v 4 ) − π 4 − (σ 2 + 2vσ + v 2 )π 2
3 2 2 3
4 4 2
− g(v + σ)ψ̄ψ − igπ ψ̄γ 5 ψ
1  1
(∂µ σ)2 + (µ2 − 3λv 2 )σ 2 + (∂µ π)2 + (µ2 − λv 2 )π 2 + ψ̄(i∂ )ψ − gv ψ̄ψ
 
=
2 2
1 2 2 λ λ λ λ
2 2
+ v µ − λv σ + µ v − λvσ 3 − σ 4 − v 4 − π 4 − σ 2 π 2 − λvσπ 2

2 4 4 4 2
− gσ ψ̄ψ − igπ ψ̄γ 5 ψ. (12)

The above can be simplified by substituting v = µ/ λ and dropping the constant terms
µ2 µ2
     
1 1 gµ
L→ (∂µ σ)2 + (µ2 − 3λ )σ 2 + (∂µ π)2 + (µ2 − λ )π 2 + ψ̄(i∂ )ψ − √ ψ̄ψ
2 λ 2 λ λ
2
 
µ µ µ µ λ λ λ
+√ µ2 − λ σ − λ √ σ 3 − λ √ σπ 2 − σ 4 − π 4 − σ 2 π 2
λ λ λ λ 4 4 2
− gσ ψ̄ψ − igπ ψ̄γ 5 ψ,
1  1
(∂µ σ)2 − 2µ2 σ 2 + (∂µ π)2 + ψ̄(i∂ )ψ − mf ψ̄ψ

=
2 2
√ √ λ λ λ
− λµσ − λµσπ − σ 4 − π 4 − σ 2 π 2 − gσ ψ̄ψ − igπ ψ̄γ 5 ψ.
3 2
(13)
4 4 2
Notice that the term linear in σ vanishes, as it should (because v is a minimum of the potential).
Furthermore, note that the fermion field has gained a mass

mf = gv = √ . (14)
λ

σ σ = i π π i
p p2 −2µ2 , p
= p2 ,

ψ ψ i
=
p p−mf

Figure 1: Feynman propagators for the Lagrangian (13).

Part (c)
The Feynman rules for the (un-renormalized) Lagrangian (13) are collected in Figs. 1 and 3.
To get the counter terms for renormalized perturbation theory we rescale the fields in (13)
by:

φi → Z φ φi ,

ψ → Z f ψ, (15)

3
Solutions to Peskin and Schroder – Andrzej Pokraka

√ √
= −6i λµ, = −2i λµ, = −6iλ,

= −6iλ, = −2iλ, = −ig,

= gγ 5

Figure 2: Feynman vertices for the Lagrangian (13).

to obtain
1 1 λ
L = Zφ (∂µ φi )2 + Zφ µ2 (φi )2 − Zφ2 ((φi )2 )2 + Zf ψ̄(i∂ )ψ − g Zφ Zf ψ̄(φ1 + iγ 5 φ2 )ψ. (16)
p
2 2 4
Defining
Zφ = 1 + δφ , Zf = 1 + δf , Zφ µ2 = µ2 + δµ , λZφ2 = λ + δλ and g Zφ Zf = g + δg ,
p
(17)
we obtain
L = L0 + δL, (18)
1 1 λ
L0 = (∂µ φi )2 + µ2 (φi )2 − ((φi )2 )2 + ψ̄(i∂ )ψ − g ψ̄(φ1 + iγ 5 φ2 )ψ, (19)
2 2 4
δφ δµ i 2 δλ
δL = (∂µ φ ) + (φ ) − ((φi )2 )2 + δf ψ̄(i∂ )ψ − δg ψ̄(φ1 + iγ 5 φ2 )ψ.
i 2
(20)
2 2 4
Substituting, φ1 = v + σ and φ2 = π we obtain the counterterm Lagrangian
δφ δµ δλ
δL = (∂µ φi )2 + (φi )2 − ((φi )2 )2 + δf ψ̄(i∂ )ψ − δg ψ̄(φ1 + iγ 5 φ2 )ψ
2 2 4
δφ δ φ δµ 2
= (∂µ σ)2 + (∂µ π)2 + v + 2vσ + σ 2 + π 2

2 2 2
δλ 4
v + 4v 2 σ 2 + σ 4 + π 4 + 4v 3 σ + 2v 2 σ 2 + 2v 2 π 2 + 4vσ 3 + 4vσπ 2 + 2σ 2 π 2


4
+ δf ψ̄(i∂ )ψ − δg ψ̄(v + σ + iγ 5 π)ψ
δφ δφ δµ 2
= (∂µ σ)2 + (∂µ π)2 + v + 2vσ + σ 2 + π 2

2 2 2
δλ 4
v + 4v σ + σ + π 4 + 4v 3 σ + 2v 2 σ 2 + 2v 2 π 2 + 4vσ 3 + 4vσπ 2 + 2σ 2 π 2
2 2 4


4
+ δf ψ̄(i∂ )ψ − δg ψ̄(v + σ + iγ 5 π)ψ
3v 2 δλ
 
δφ 2 δφ 2 3 δµ
σ2

= (∂µ σ) + (∂µ π) + δf ψ̄(i∂ )ψ + vδµ − v δλ σ + −
2 2 2 2
v 2 δλ
 
δµ
+ − π 2 − vδg ψ̄ψ − δg σ ψ̄ψ − vδλ σ 3 − vδλ σπ 2 − δg π ψ̄iγ 5 ψ
2 2
δλ δλ δλ v 2 δµ v 4 δλ
− σ2 π2 − σ4 − π4 + − . (21)
2 4 4 2 4

4
Solutions to Peskin and Schroder – Andrzej Pokraka

 
= i vδµ − v 3 δλ , = i δφ p2 − δµ − 3v 2 δλ ,


= i δφ p2 − δµ − v 2 δλ ,  − vδg ) ,
= i (δf p

= −6ivδλ , = −2ivδλ ,

= δg γ 5 , = −6iδλ ,

= −6iδλ , = −2iδλ ,

Figure 3: Feynman vertices for the counterterm Lagrangian (21).

The corrections to the mass are given by δmf = Σ(p)|  p=mf . Thus, we need to calculate δg
and any radiative corrections to v. We are to use renormalization conditions such that g and v
do not receive any radiative corrections. This fixes δg , which must satisfy the equation
p
 
 
ū(p)  u(p) = gū(p)γ 5 u(p). (22)
 

 
p
amputated

More specifically, δg is fixed by the condition


ū(p)δg γ 5 u(p) = ū(p))V (p, p)γ 5 u(p) (23)
where
p0 p0 p0 p0

V (p, p0 )γ 5 = + + + . (24)

p p p p
Evaluating the sum of the first two diagrams above we obtain

dd k i 0
Z
5 i(p  + k + mf ) (gγ 5 ) i(p + k + mf ) (gγ 5 )γ 5
V12 (p, p0 ) = (gγ ) 2
d
(2π) k 2 0 2
(p + k) − mf (p + k)2 − m2f
dd k 0
Z
i i(p
 + k + mf ) (gγ 5 ) i(p
 + k + mf ) (−ig)γ 5
+ (−ig) 2
d 2
(2π) k − 2µ 2 0 2
(p + k) − mf (p + k)2 − m2f
!
dd k 0
− k + mf ) (p 0
− k + mf )
Z
3 1 (−p + k + mf ) 1 (p + k + mf ) (−p
= −ig   − 2  
(2π)d k 2 (p0 + k)2 − m2f (p + k)2 − m2f k − 2µ2 (p0 + k)2 − m2f (p + k)2 − m2f
(25)

5
Solutions to Peskin and Schroder – Andrzej Pokraka

Setting p0 = p and p2 = m2f , V12 simplifies to

dd k
Z  
1 1 1
V (p, p) = −ig 3 − 2 . (26)
(2π)d k2 k − 2µ2 (p + k)2 − m2f

To finish simplifying V12 (p, p) we need to evaluate two integrals of the form

dd k
Z
1
I(M ) ≡
(2π) (k − M )((p + k)2 − m2f )
d 2 2

dd k
Z Z
1
= dx
(2π) (k − (1 − x)M + 2xp · k + xp2 − xm2f )2
d 2 2

dd k
Z Z
1
= dx
(2π)d ((k + xp)2 − x2 p2 − (1 − x)M 2 + xp2 − xm2f )2
dd `
Z Z
1
= dx
(2π)d (`2 − ∆)2
Z  2−d/2
i 1
= dx d/2
Γ(2 − d/2)
(4π) ∆
Z  2−d/2
i 4π
= dx Γ(2 − d/2) (27)
16π 2 ∆

where ∆(M ) = x2 p2 + (1 − x)M 2 − xp2 + xm2f = x2 m2f + (1 − x)M 2 . Using I, we obtain


 √ 
V12 (p, p) = −ig 3 I(0) + I( 2µ)
g3
Z       
4π 4π 2 1
= dx 1 +  log − 1 −  log √ + O( ) + γE + O()
16π 2 ∆(0) ∆( 2µ) 
g3
Z      
4π 4π
= dx log − log √ + O() (1 + O())
16π 2 ∆(0) ∆( 2µ)
!
g3 x2 m2f + 2(1 − x)µ2
Z
= dx log . (28)
16π 2 x2 m2f

Next we evaluate the last two diagrams in V ,


"
0
Z
dd k i(k + mf ) i √ i
V34 (p, p ) = (gγ 5 ) 2 2 (−ig) (−2iµ λ)
(2π) d k − mf 2
(k + p) − 2µ 2 (k + p0 )2
#
i(k + mf ) 5 i √ i
(−ig) 2 (−igγ ) (−2iµ λ) γ5
k − m2f (k + p)2 (k + p0 )2 − 2µ2
" #
2
√ Z dd k (−k + mf ) (k + mf )
= 2g µ λ + 2
(2π)d (k 2 − m2f )((k + p)2 − 2µ2 )(k + p0 )2 (k − m2f )(k + p)2 ((k + p0 )2 − 2µ2 )
√ dd ` (−` + y p 0 0
 
 + mf ) + (` − y p − zp
Z Z Z
= 2g 2 µ λΓ(3) dy dz  + zp  + mf )
(2π)d (`2 − ∆1 )3 (`2 − ∆2 )3
√ dd ` y p 0 0
 
− mf
Z Z Z
2 + zp + mf yp + zp
= 4g µ λΓ(3) dy dz   −   (29)
(2π)d (`2 − ∆1 )3 (`2 − ∆2 )3

where
2
∆1 = −y(1 − y)p2 − z(1 − z)p0 + 2yzp · p0 + (1 − y − z)m2f − 2yµ2 , (30)
02
∆2 = −y(1 − y)p2 − z(1 − z)p + 2yzp · p0 + (1 − y − z)m2f − 2zµ2 . (31)

6
Solutions to Peskin and Schroder – Andrzej Pokraka

Setting p0 = p and p2 = m2f , V34 simplifies to

√ dd `
 
 − mf
Z Z Z
(y + z)p
 + mf − (y + z)p
V34 (p, p) = 2g 2 µ λΓ(3) dy dz
(2π)d (`2 − ∆1 |p0 =p )3 (`2 − ∆2 |p0 =p )3

 
 − mf
Z Z
i (y + z)p + mf − (y + z)p
= 2g 2 µ λ dy dz
16π 2 ∆1 |p0 =p ∆2 |p0 =p

 
 − mf
Z Z
i (y + z)p + mf − (y + z)p
= 2g 2 µ λ dy dz
16π 2 ∆1 |p0 =p ∆1 |p0 =p
√ i
Z 1 Z 1−x
mf
= 4g 2 µ λ dx dy
16π 2 0 0 (1 − 2x − 2y + 2xy + x2 + y 2 )m2f − 2xµ2
m2f
Z 1 Z 1−x
i
= 4gλ dx dy (32)
2
16π 0 0 (1 − 2x − 2y + 2xy + x2 + y 2 )m2f − 2xµ2

The counterterm, δg , is then fixed to be

δg = −V (p, p)
= −V12 (p, p) − V34 (p, p)
Z 1 " !
g 2
x2 m2f + 2(1 − x)µ2
=− dx g log
16π 2 0 x2 m2f
#
m2f
Z 1−x
+ 4λi dy (33)
0 (1 − 2x − 2y + 2xy + x2 + y 2 )m2f − 2xµ2

which is finite.
Next we calculate Σ(p).
 The self-energy is given by

0
− iΣ(p)
 = −iΣ (p)
 + , (34)

where

− iΣ0 (p)
 = + . (35)

The primed self energy is


" #
dd k
Z
0 5 i(k+p + mf ) 5 i i(k + p + mf ) i
−iΣ (p)
 = (gγ )  (gγ ) 2 + (−ig)  (−ig) 2
(2π)d (k + p)2 − m2f k (k + p)2 − m2f k − 2µ2
" #
dd k (−k − p
Z
= −g 2  + mf ) 1 − (k + p  + mf ) 1
(2π)d (k + p)2 − m2f k 2 (k + p)2 − m2f k 2 − 2µ2
" #
dd k (−k − p
Z Z
2 + mf ) (k + p + mf )
= −g dx  − 2 
(2π)d (k 2 + 2xk · p + xp2 − xm2f )2 (k + 2xk · p + xp2 − xm2f − 2(1 − x)µ2 )2
" #
dd k − −
Z Z
2 (−` + xp p + m f ) (` xp + p + m f )
= −g dx − 2
    
(2π)d (`2 − x2 p2 + xp2 − xm2f )2 (` − x2 p2 + xp2 − xm2f − 2(1 − x)µ2 )2
dd k (−(1 − x)p
 
+ mf ) ((1 − x)p
Z Z
2 + mf )
= −g dx  −  (36)
(2π)d (`2 − ∆1 )2 (`2 − ∆2 )2

7
Solutions to Peskin and Schroder – Andrzej Pokraka

where ∆1 = −x(1 − x)p2 + xm2f and ∆2 = ∆1 + 2(1 − x)µ2 . Evaluating the momentum integrals
we obtain
Z      
0 2 i 4π 4π
−iΣ (p) = −g (4π)2 dx (−(1 − x)p + mf ) ∆1 − ((1 − x)p
 + m f ) ∆2 Γ()

4πe−γE 4πe−γE
    
−2(1 − x)p
Z
2 i ∆2
= −g dx  + mf log − (1 − x)p log
(4π)2  ∆1 ∆2 ∆1
(37)

8
Solutions to Peskin and Schroder – Andrzej Pokraka

Problem 12.3: Gross-Neveu model.


The Gross-Neveu model is a model in two spacetime dimensions of fermions with a discrete
chiral symmetry:
1
L = ψ̄i i∂ ψi + g 2 (ψ̄i ψi )2 (38)
2
with i = 1, . . . , N . The kinetic term of tow-dimensional fermions is built form matrices γ µ that
satisfy the two-dimensional Dirac algebra. These matrices can be 2 × 2:

γ 0 = σ2 , γ 1 = iσ 1 , (39)

where σ i are the Pauli matrices. Define

γ 5 = γ 0 γ 1 = σ3 ; (40)

this matrix anti commutes with the γ µ .

(a)
(b)
(c)
(d)

(e)
(f)

You might also like